Legendre Transformation of f(x) = x^3

Click For Summary
The discussion focuses on finding the Legendre transformation of the function f(x) = x^3. The key steps involve calculating the derivative m(x) = f'(x) = 3x^2 and expressing x in terms of m. The transformation is expressed as g = f(x) - xm, leading to g(m) = x^3 - x(3x^2). Participants note confusion regarding the correct formulation of the transformation and the distinction between maximizing and minimizing functions. The conversation highlights the differences in approaches between various texts and fields, particularly in physics versus mathematics.
RJLiberator
Gold Member
Messages
1,094
Reaction score
63

Homework Statement


[/B]
Find the Legendre Transformation of f(x)=x^3

Homework Equations


m(x) = f'(x) = 3x^2
x = {\sqrt{\frac{m(x)}{3}}}
g = f(x)-xm

The Attempt at a Solution



I am reading a quick description of the Legendre Transformation in my required text and it has the example giving for the function f(x) = \frac{1}{2}e^{2x}

Thus, I am trying to follow through with it as follows:

g=f(x)-xm = x^3-x(3x^2)

The problem for me exists in the next step. For them, they had an easy simplification for their example problem. In my problem, m does not equal x^3, m is equal to 3x^2.

Can I do this to solve the transformation:

Since x = {\sqrt{\frac{m(x)}{3}}}

We have
({\sqrt{\frac{m(x)}{3}}})^3- {\sqrt{\frac{m(x)}{3}}}m = g(m)

And that is the Legendre transformation of x^3.

Note* First time ever being exposed to Legendre Transformation.
 
Physics news on Phys.org
RJLiberator said:

Homework Statement


[/B]
Find the Legendre Transformation of f(x)=x^3

Homework Equations


m(x) = f'(x) = 3x^2
x = {\sqrt{\frac{m(x)}{3}}}
g = f(x)-xm

The Attempt at a Solution



I am reading a quick description of the Legendre Transformation in my required text and it has the example giving for the function f(x) = \frac{1}{2}e^{2x}

Thus, I am trying to follow through with it as follows:

g=f(x)-xm = x^3-x(3x^2)

The problem for me exists in the next step. For them, they had an easy simplification for their example problem. In my problem, m does not equal x^3, m is equal to 3x^2.

Can I do this to solve the transformation:

Since x = {\sqrt{\frac{m(x)}{3}}}

We have
({\sqrt{\frac{m(x)}{3}}})^3- {\sqrt{\frac{m(x)}{3}}}m = g(m)

And that is the Legendre transformation of x^3.

Note* First time ever being exposed to Legendre Transformation.

The Legendre Transform ##f^*(p)## of the convex function ##f(x)## with domain ##x \geq 0## is
$$f^*(p) = \max_{x \geq 0} [p x - f(x)] $$
(or with ##\sup## replacing ##\max## if necessary..

Note that you SUBTRACT ##f(x)##, not add it, and you need another variable ##p## as the argument of the transformed function.

Everything would fall apart if you tried to maximize ##f(x) - px##; that is, if you added ##f## instead of subtracting it, the whole theory would fail. There are good reasons for that, but never mind them for now.
 
  • Like
Likes RJLiberator
I am following an example laid out by the book, where am I going wrong? Is it due to the concave vs. convex nature?

and you need another variable p as the argument of the transformed function.

Would my "m" variable be equal to your "p" variable?

So, if I follow the path of mx-f(x), I get:

g(m) = mx-f(x)= x(3x^2)-x^3
And so:
g(m) = {\sqrt{\frac{m(x)}{3}}}m-({\sqrt{\frac{m(x)}{3}}})^3
 
RJLiberator said:
I am following an example laid out by the book, where am I going wrong? Is it due to the concave vs. convex nature?
Would my "m" variable be equal to your "p" variable?

So, if I follow the path of mx-f(x), I get:

g(m) = mx-f(x)= x(3x^2)-x^3
And so:
g(m) = {\sqrt{\frac{m(x)}{3}}}m-({\sqrt{\frac{m(x)}{3}}})^3

Yes, my "p" is your "m".

Some people use ##f(x) - mx## and minimize it instead of maximizing, but the max form is pretty standard. I think it is often done differently in Physics than in Mathematics, and apparently your un-named book does it differently.

I cannot make any sense of what you did; your formula for the Lagrange-transformed function ##g(m)## should not have an "##x##" in it, and the final answer should be negative.
 
I will continue to look into the problem and with your added information I'll try to make sense of it all.
apparently your un-named book does it differently.
Statistical and Thermal Physics:
With Computer Applications
Harvey Gould & Jan Tobochnik

 
Question: A clock's minute hand has length 4 and its hour hand has length 3. What is the distance between the tips at the moment when it is increasing most rapidly?(Putnam Exam Question) Answer: Making assumption that both the hands moves at constant angular velocities, the answer is ## \sqrt{7} .## But don't you think this assumption is somewhat doubtful and wrong?

Similar threads

Replies
4
Views
2K
  • · Replies 10 ·
Replies
10
Views
2K
  • · Replies 5 ·
Replies
5
Views
2K
  • · Replies 10 ·
Replies
10
Views
2K
  • · Replies 1 ·
Replies
1
Views
2K
  • · Replies 3 ·
Replies
3
Views
899
  • · Replies 5 ·
Replies
5
Views
2K
  • · Replies 7 ·
Replies
7
Views
2K
Replies
2
Views
1K
  • · Replies 5 ·
Replies
5
Views
2K